Oral Polytechnique (niveau PC)

Bonjour,
j'essaie de traiter l'exercice suivant :
Soit $f : x \mapsto \int_x^{+\infty} \frac{e^{-t^2}}{t}dt$.
1. Domaine de définition de f ? f est-t-elle dérivable ?
2. Montrer que $x \mapsto e^{x^2} f(x)$ est bornée sur $[1; +\infty[$
3. Donner un équivalent de $f(x)$ en $0^+$.
4. L'intégrale $\int_0^{+\infty} f(x)dx$ est-elle convergente ?

1. Je trouve que $f$ est définie et dérivable sur $\R_+^* = ]0; +\infty[$ et que $f'(x) = - \frac{e^{-x^2}}{x}, \forall x \in \R_+^*$.\
2. Je note $\phi : x \mapsto e^{x^2} f(x)$, dérivable sur $[1; +\infty[$. En dérivant, on obtient :
$\phi'(x) = \frac{2x^2 \phi(x) - 1}{x}$. En suite je ne vois pas comment étudier le signe de ${2x^2 \phi(x) - 1}$ pour en déduire les variations de $\phi$ sur $[1; +\infty[$. Une idée ?

3. Pour l'équivalent de $f(x)$ en 0, mon intuition me dit $-ln(x)$ mais je vois pas trop comment le montrer (si c'est vrai ?)
4. Avec un tel équivalent , $f$ serait intégrable sur $]0;1]$ et la question 2. fournit qu'au voisinage de $+\infty$ $f(x)$ est dominé par $e^{-x^2}$ qui est intégrable donc par comparaison de fonctions continues et de signe constant, $f$ est intégrable sur $[1; +\infty[$ et l'intégrale $\int_0^{+\infty} f(x)dx$ converge. Ce dernier point est-il bon ?
Merci d'avance!

Réponses

  • pour la 2) faire une IPP donne la solution...
    pour 3), tu peux intégrer l'équivalent qui est positif dans un voisinage de $0$ (cf sommation des relations de comparaison)
    pour 4), $f$ est CPM et dominée par une fonction intégrable sur $\mathbb{R}^{+}$, elle donc intégrable sur $\mathbb{R}^{+}$ (oui, ta justification est bonne mais tu peux faire un peu plus vite!)
  • Pour le 3) tu peux aussi étudier $f(x) + \log(x)$ au voisinage de $0$ et montrer que c'est un $o(\log x)$.
  • @BobbyJoe
    En effectuant une IPP, je trouve : $\quad\displaystyle \phi(x) = \frac{1}{2x^2} - e^{-x^2} \int_x^{+\infty} \frac{e^{-t^2}}{t^3} dt$.
    En quoi cela m'aide-t-il à montrer que $\phi$ est bornée sur $[ 1; +\infty [$ ?
  • @ Fabrice2
    Questions posées de façon claire, avec un énoncé correct, en précisant le niveau requis, excellent.
    Pour la question 2, il n'est pas nécessaire d'étudier les variations d'une fonction, il suffit d'utiliser le fait que $x \geq 1$, d'où $1/t \leq t$.
    Pour la question 3, ceux qui veulent donner des conseils utiles devraient s'enquérir du programme de la classe.
    En allant voir ici :
    <https://www.scei-concours.fr/CPGE/BO/Mathematiques_PC.pdf&gt;
    on constatera que ni la sommation ni l'intégration des relations de comparaison ne sont au programme de PC. Les évoquer ne rend donc pas service au questionneur.
    Mais comme c'est une intégrale entre $x$ et $+ \infty$, il n'est pas nécessaire d'utiliser cette propriété.
    Bon courage.
    Fr. Ch.
  • @Fabrice2
    Avec une ipp
    $\quad\displaystyle \phi(x) = \frac{1}{2x^2} - e^{x^2} \int_x^{+\infty} \frac{e^{-t^2}}{t^3} dt$
    et tu peux majorer $e^{-t^2}$ sur $[x,+\infty[$ par $e^{-x^2} $
    donc $\quad\displaystyle |\phi(x) |\leq |\frac{1}{2x^2}|+e^{x^2}\int_x^{+\infty} \frac{e^{-x^2}}{t^3} dt\leq \frac 12 +\int_1^{+\infty} \frac{1}{t^3} dt<+\infty
    $
    @Chaurien
    Je n'ai pas compris ta suggestion pour la 2, peux-tu expliquer d'avantage?
    Le 😄 Farceur


  • @ Fabrice2
    Et ta solution de la question 4 est la plus correcte, inutile de chercher midi à quatorze heures.
    Pour la question 3, tu as eu une excellente intuition. Pour la prouver, introduis de force ton log :
    $f(x)=\int_{x}^{+\infty }\frac{e^{-t^{2}}}{t}dt=\int_{x}^{1}\frac{1}{t}dt+\int_{x}^{1}\frac{e^{-t^{2}}-1}{t}dt+\int_{1}^{+\infty }\frac{e^{-t^{2}}}{t}dt$.
    Bien cordialement,
    FR. Ch.
  • @gebrane0
    Je veux bien expliquer davantage :
    Si $t\geq x\geq 1$, alors $0<\frac{1}{t}\leq 1 \leq t$, d'où :
    $f(x)=\int_{x}^{+\infty }\frac{e^{-t^{2}}}{t}dt\leq \int_{x}^{+\infty}e^{-t^{2}}tdt=[-\frac{1}{2}e^{-t^{2}}]_{t=x}^{t\rightarrow +\infty }=\frac{1}{2}e^{-x^{2}}$.

    Bonne journée

    Fr. Ch.

    [small]Elle avait peu d'avantages
    Pour en avoir davantage
    Elle s'en fit rajouter
    A l'institut de beauté, ah ah ah ![/small]

  • Quelques remarques supplémentaires.
    1. En $0^+$, on démontre un peu plus que l'équivalent, on prouve en fait que $f(x)=\int_{x}^{+\infty }\frac{e^{-t^{2}}}{t}dt =- \ln x+C+o(1)$.
    2. Dans l'exercice tel qu'il est posé, aucune Intégration Par Parties (IPP) n'est nécessaire.
    3. En $+ \infty$ on pouvait demander un peu plus que la majoration, on pouvait demander un équivalent « simple », et là l'IPP est indispensable, et comme le montre Gebrane, il n'est pas nécessaire d'appliquer l'intégration des relations de comparaison, hors-programme en PC, car l'intégrale est sur $[x, +\infty[$.
    4. On peut aller un peu plus loin que l'intégrabilité de $f$ sur $[0, +\infty[$, et démontrer que : $\int_{0}^{+\infty }f(x)dx=\int_{0}^{+\infty }e^{-t^{2}}tdt$.
    Le collègue qui a posé ça a manqué de souffle.
    Bonne soirée.
    Fr. Ch.
    28/09/2017
  • On peut facilement généraliser avec $f(x)=\displaystyle \int_{x}^{+\infty }\dfrac{e^{-t^{\alpha}}}{t^{\beta}}dt$ pour $\alpha>0$ et $\beta\in \R$.

    $f$ est intégrable sur $]0,+\infty[$ si et seulement si $\beta<2$.

    On a alors $\displaystyle\int_{0}^{+\infty }f(x)dx=\int_{0}^{+\infty }\dfrac{e^{-t^{\alpha}}}{t^{\beta-1}}dt=\dfrac1{\alpha}\Gamma\left(\dfrac{2-\beta}{\alpha}\right)$.
  • Merci à tous pour votre aide !
    @Chaurien,
    Pouvez-vous expliquer davantage comment obtenir un équivalent simple de $\phi(x)$ à partir de l'IPP faite en 2. ?
    Merci d'avance
  • [Chaurien écrivait : http://www.les-mathematiques.net/phorum/read.php?4,1531530,1534786#msg-1534786
    [Inutile de recopier un message présent sur le forum. Un lien suffit. AD]

    Oula, y a visiblement un problème de primitive là
Connectez-vous ou Inscrivez-vous pour répondre.